Please confirm topic selection

Are you sure you want to trigger topic in your Anconeus AI algorithm?

Please confirm action

You are done for today with this topic.

Would you like to start learning session with this topic items scheduled for future?

Review Question - QID 213961

In scope icon L 1 A
QID 213961 (Type "213961" in App Search)
A 40-year-old male presents to your clinic for evaluation of 8 months of left shoulder pain and weakness after a fall while skiing. MRI studies are shown in Figures A and B. Which of the following physical exam findings would be highly probable in this patient?
  • A
  • B
  • A
  • B
Attach Treatment Poll
Treatment poll is required to gain more useful feedback from members.
Please enter Question Text
Please enter at least 2 unique options
Please enter at least 2 unique options
Please enter at least 2 unique options